I feel silly for getting this two star flaw question wrong in timed and in BR lol. I actually pre-phrased both answers which is why I found this one so difficult.
When I read the stimulus the first issue I noted was that we have no context ...
Does someone mind explaining why D can't be right? I think A makes sense but also feel like paranoid, in the stimulus, is a condition that keeps changing along with changes in society.
Maybe it's because it doesn't **most** strongly support ...
Is anyone able to explain this question to me? I am horrible with proportions/math/etc, and I don't understand why my answer was wrong or why the right one is right. Thank you!!
#Help
... accident, that PT 26 S1 Game 2 and PT 81 S4 Game 2 ... because I had randomly done PT 26 S1 game 2 as ... a warmup LG before taking PT 81 today. They did ask ...
I selected D, but the correct answer is E. Can someone help me understand why D is wrong?
If I put Gina in the Wednesday slot, I am wondering why this pattern won't work:
HJGHFJ
Struggling to see what rule I am violating. Thanks.
Can someone explain this necessary assumption problem to me? I got the question wrong the first time and during blind review (I answered B, then E). The answer doesn't seem to be A or C, so it must be D. But why?
... am really confused I think PT1 S3 Q07 has answer ... are three components 1,2, 4
( 1,2 both belong ... given us quantity comparison of 1,2,4 they are ... increase in greater proportion than 1 because the ratio of ... A/B= west/east 1=young male 2 =old male ...
I'm trying to identify flaws. is PT 56 S3 Q10 an equivocation flaw; can you use more than a word but a concept in this type of flaw?
Is PT 52 S3 Q4 a false appeal flaw?
Is PT 54 S4 Q16 an implication flaw ?
thanks
I figured since there is no explanation video on this one, I'd offer my two cents. Please feel free to respond if my explanation is lacking anything or if you have a better one.
So the correct AC is B and here is why. The stimulus tells us ...